Difference between revisions of "2017 AMC 12B Problems/Problem 1"
Toffeecube (talk | contribs) (Created page with "==Problem 1==") |
Toffeecube (talk | contribs) (→Problem 1) |
||
Line 1: | Line 1: | ||
==Problem 1== | ==Problem 1== | ||
+ | |||
+ | Kymbrea's comic book collection currently has <math>30</math> comic books in it, and she is adding to her collection at the rate of <math>2</math> comic books per month. LaShawn's collection currently has <math>10</math> comic books in it, and he is adding to his collection at the rate of <math>6</math> comic books per month. After how many months will LaShawn's collection have twice as many comic books as Kymbrea's? | ||
+ | |||
+ | <math>\textbf{(A)}\ 1\qquad\textbf{(B)}\ 4\qquad\textbf{(C)}\ 5\qquad\textbf{(D)}\ 20\qquad\textbf{(E)}\ 25</math> | ||
+ | |||
+ | ==Solution== |
Revision as of 17:35, 16 February 2017
Problem 1
Kymbrea's comic book collection currently has comic books in it, and she is adding to her collection at the rate of comic books per month. LaShawn's collection currently has comic books in it, and he is adding to his collection at the rate of comic books per month. After how many months will LaShawn's collection have twice as many comic books as Kymbrea's?